Maximizing Limit Values for Exponential Functions with Variable Parameters

  • MHB
  • Thread starter karush
  • Start date
In summary, the limit of $\left(1+\dfrac{a}{x}\right)^{bx}$ as $x$ approaches infinity can be simplified to $e^{ab}$ using L'Hopital's rule. The variables $a$ and $b$ are treated as constants, and the expression inside the parentheses can be simplified using the natural logarithm and exponential functions.
  • #1
karush
Gold Member
MHB
3,269
5
$\displaystyle\lim_{{x}\to{\infty}}\left(1+\dfrac{a}{x}\right)^{bx}$

Ok I am a little stumped already because we have 3 variables in this a,x and b

W|A returned $e^{ab}$ but not what the steps are, maybe next...

$\displaystyle bx\lim_{{x}\to{\infty}}\left(1+\dfrac{a}{x}\right)$
 
Last edited:
Physics news on Phys.org
  • #2
karush said:
$\displaystyle\lim_{{x}\to{\infty}}\left(1+\dfrac{a}{x}\right)^{bx}$

Ok I am a little stumped already because we have 3 variables in this a,x and b

note that $a$ and $b$ represent constants, and ...

$\displaystyle\lim_{{x}\to{\infty}}\left(1+\dfrac{1}{x}\right)^x = e$

$\displaystyle\lim_{{x}\to{\infty}}\left(1+\dfrac{a}{x}\right)^x = e^a$

... so, what happens to the limit with the constant "$b$" thrown in as an outer exponent?
 
  • #3
not sure if this is the correct way to say it
but are they not just scalarsconcering what is inside the () can that be distributed or added together
 
  • #4
karush said:
not sure if this is the correct way to say it
but are they not just scalars

The limit is with respect to $x$, so $a$ and $b$ are treated as constants.

Getting things into a form where we can apply L'Hopital's rule,



\(\displaystyle \]\exp\left(\frac{\log\left(1+\frac ax\right)}{\frac{1}{bx}}\right)\)



Now *differentiate and simplify* inside the brackets; you'll end up with $e^{ab}$.
 
  • #5
Greg said:
\(\displaystyle \]\exp\left(\frac{\log\left(1+\frac ax\right)}{\frac{1}{bx}}\right)\)

ok I'm not familiar with "exp"
 
  • #6
karush said:
ok I'm not familiar with "exp"
exp is the same as e:
\(\displaystyle exp( \theta ) = e^{ \theta }\)

-Dan
 
  • #7
made my day :cool:
 

1. What is "6.6.60 limiit possible L'H"?

"6.6.60 limiit possible L'H" refers to the mathematical concept of using L'Hopital's rule to find the limit of a function as it approaches a certain value, specifically when the value is 6.6.60.

2. How do you use L'Hopital's rule to find a limit?

To use L'Hopital's rule, you must first have a function in the form of a fraction where both the numerator and denominator approach 0 or infinity as the variable approaches the limit value. Then, you can take the derivative of both the numerator and denominator and evaluate the limit again. This process can be repeated until a definitive answer is reached.

3. What is the significance of 6.6.60 in this context?

6.6.60 is the specific value that the limit is approaching. This value can be any number, but in this case, it is 6.6.60.

4. Is L'Hopital's rule always applicable for finding limits?

No, L'Hopital's rule can only be used when the limit is in an indeterminate form, such as 0/0 or infinity/infinity. In other cases, different methods must be used to find the limit.

5. Can L'Hopital's rule be used for all types of functions?

No, L'Hopital's rule is typically used for rational functions (functions in the form of a fraction), but it can also be applied to other types of functions as long as they are in an indeterminate form at the limit value.

Similar threads

  • Calculus
Replies
2
Views
1K
Replies
8
Views
1K
  • Calculus
Replies
3
Views
1K
  • Calculus
Replies
3
Views
938
Replies
2
Views
1K
  • Calculus
Replies
1
Views
1K
Replies
16
Views
2K
Replies
2
Views
943
Back
Top